Você está na página 1de 7

Assignment III

FUNCTIONAL ANALYSIS


July 4, 2016

Submitted to

Dr. Shiferaw Feyissa

Name
1.
2.
3.
4.
5.

Abener Tewodros .
Hana Endiris . . .
Habtamu Meressa
Miliyon Tilahun . .
Sisai Bekele . . . .

.
.
.
.
.

.
.
.
.
.

.
.
.
.
.

.
.
.
.
.

.
.
.
.
.

.
.
.
.
.

.
.
.
.
.

.
.
.
.
.

.
.
.
.
.

.
.
.
.
.

.
.
.
.
.

.
.
.
.
.

.
.
.
.
.

.
.
.
.
.

.
.
.
.
.

.
.
.
.
.

.
.
.
.
.

.
.
.
.
.

.
.
.
.
.

.
.
.
.
.

.
.
.
.
.

ID No
. GSR/1417/08
. GSR/1419/08
. GSR/1399/08
. GSR/1401/08
. GSR/1426/08

Functional Analysis Problems


1. Let X be a finite dimensional normed space and Y be normed space.
Show that
(a) X is a Banach space.
(b) Every linear operator T : X Y is continuous.
Proof. (a) Suppose (X, k k) is the normed space, and (ei )N
i=1 is a basis
PN
for X. For x = j=1 j ej , we can then define
v
uN
uX
0
kxk = t
|j |2
j=1

whence k k0 : X R is a norm for X. Since all norms on a finite


dimensional normed space are equivalent, there is a constant C > 0
such that
1
kxk0 kxk Ckxk0 , x X.
C
To prove that X is a Banach space, let x1 , x2 , . . . be a Cauchy sequence
in (X, k k). That is, for all > 0 there is an M 1 such that
kxj xk k < , for all j, k M.
Let us write each xk in this sequence in the basis (ej ) as xk =
for some constants k,j C. For k, l 1 we then have
kxk xl k

PN

j=1

k,j ej

1
kxk xl k0
Cv
uN
X
1u
t
|k,j l,j |2
C j=1

1
|k,j l,j |
C

for all j = 1, . . . , N . It follows that (k,1 )


k=1 , . . . , (k,N )k=1 are Cauchy
sequences in C. As C is complete,
these converge to some complex
PN
numbers 1 , . . . , N . Let x = j=1 j ej .

For each k = 1, 2, . . ., we then have


kx xk k

Ckx xk k0
v
uN
uX
|j k,j |2 .
Ct
j=1

By taking k it follows that (xj ) converges to x V .


(b) Let e1 , . . . , en be a basis of X. Then for x X we have
X
X
X
kT xkY = kT
i ei kY = k
i T ei kY
|i |kT ei kY
i

Let > 0 be given and let M = maxi kT ei kY . Let = M


. Then for x
with kxk, we have
X
X
kT xkY
|i |kT ei kY M
|i | <
i

Hence T is continuous.
R1
2. Let X = C[0, 1] with norm kf k1 = 0 |f (t)|dt and Y = C[0, 1] with
norm kf k = supt[0,1] |f (t)|. Define K : C[0, 1] C[0, 1] R by
R1
K(f ) = 0 K(s, t)f (t)dt, s [0, 1]. Show that K is a continuous linear
map.
Solution.

3. Let T : X R be a linear functional such that T is non-zero. Show


that the following statements are equivalent.
(a) T is continuous.
(b) The nullity of T is closed.
Solution. (a b) If T is continuous, then the inverse image of closed
set under T is closed. Hence
ker T = T 1 [{0}]
is closed.
(a b) Assume T 6= 0 and T 1 [{0}] is closed. Pick e X such that
f (e) = 1.
Suppose by way of contradiction kT k = . Then there exists a sequence
{xn } in X with kxn k = 1 and T (xn ) n for all n. Note that the
xn
sequence yn defined by yn = e T (x
, satisfies yn T 1 [{0}] for all n
n)
and yn e. Since the set is closed it follows that e must belong to it
and consequently T (e) = 0 which is a contradiction.
4. Let c = {xn } be the set of all sequences in R such that limn xn
exists. Show that c is a closed subspace of ` and hence is a Banach
space.
(n)

Solution. For each n, let n = {xk }k0 is in c and hence convergent.


(n)
Let say that n = {xk }k0 converges to un R. Now
|um un | km n k ,

m, n N

So since {n }n0 is a Cauchy sequence in c, it follows that {un }n0 is a


Cauchy sequence in R, so there exists u R such that limn un = u.
Claim = {xk }k0 converges to u. To see this
(n)

(n)

|xk u| |xk xk | + |xk un | + |un u|


(n)

k n k + |xk un | + |un u|
Let > 0. Choose n0 N such that |un0 u| < 3 and k n0 k < 3 .
(n )
Choose K N such that |xk 0 un0 | < 3 whenever k > K. Then for
k K all three terms on the right hand side of the above equation are
less than 3 , and thus |xk u| < .
Thus xk u. Therefore = {xk }k0 converges so c and hence c is
a Banach subspace of ` .

5. Show that the dual space of `p is `q where

1
p

1
q

= 1.

Solution.
6. Let T be a self adjoint bounded linear operator in a Hilbert space H.
Prove that
kT k = sup |hx, T (x)i|
kxk=1

Solution.
7. Let H be a Hilbert space and fy (x) = hy, xi be a bounded linear functional. Prove that kfy k = kyk.
Solution.

8. Show that T : R R defined by T (x) = 2 + x tan1 x has no fixed


point and |T (x) T (y)| < |x y| for all x, y R.
Solution. We have

T x x = + x arctan(x) x
2

= arctan(x) > 0
(since /2 < arctan(x) < /2)
2
So T has no fixed point.
Since
T 0 (x) =

x2
1 + x2

we have 0 < T 0 < 1. The the Mean Value Theorem implies that if x 6= y
T x T y = T 0 ()(x y)
for some between x and y, so
|T x T y| < |x y|

9. Let X and Y be normed space. If T1 : X Y is a closed linear operator


and T2 : X Y be a bounded linear operator then show that T1 + T2
is closed linear operator.
Solution. Let xn X, and x X, y Y such that as n
xn x in X
and
(T1 + T2 )xn = T1 xn + T2 xn y

in Y

Since T2 is bounded there exists


T2 x = lim T2 xn
n

and so T1 xn y T2 x.
The closedness of T1 implies that x X and T1 x = y T2 x that is
(T1 + T2 )x = T1 x + T2 x = y.

10. If T is a contraction then show that T n where n is a positive integer is


a contraction. If T n is a contraction for n > 1 show that T may not be
a contraction.
Solution.

Você também pode gostar